Download as pdf or txt
Download as pdf or txt
You are on page 1of 9

10 Electrostatics

®
Where r1 is the position vector of point A and r®i is the position # Illustrative Example 1.2
vector of the vertex due to which force on charge at A is given
by the force vector given in above equation-(1.3) Two particles, each of mass of 5 gm and charged with charge
1.0 × 10–7 C each stay in equilibrium at the verge of sliding on a
horizontal table with a separation of 10 cm between them. The
Z
coefficient of friction m between each particle and the table is
(a, 0, a) 1
the same. Find m.
4
Solution
(a , a, a) (0, a, a)
3 2
Figure-1.24 shows the situation described in the problem. The
forces acting on A are coulombic force and frictional force and
A (0, 0, 0)
at the limiting equilibrium limiting friction acting on it will be
5
equal to the coulombic force by other particle which are given
X (a, 0, 0)
as
(0, a, 0)
6 Kq 2 9 ´109 ´ (10 -7 ) 2
(a, a, 0) 7 Fc = = = 9 × 10–3 N
Y r2 (10 ´ 10 -2 ) 2
and f = µN = µmg = µ (5 × 10–3 × 10) = µ (5 × 10–2) N
Figure 1.23
q = 10–7 C q = 10–7
From the figure-1.23 the different forces acting on A are given FC f f FC
5 gm 5 gm
as A 10 cm B

® Kq (-akˆ)
2
Figure 1.24
FA1 = 3
a
For equilibrium, we have
® Kq (-ajˆ - akˆ)
2 Fc = f
FA 2 =
( 2 a )3
Þ 9 × 10–3 = µ (5 × 10–2)
9 ´10 -3
® Kq 2 (-aiˆ - ajˆ - akˆ) Þ µ= = 0·18
F A3 = 3 5 ´10 -2
( 3a)
# Illustrative Example 1.3
® Kq 2 (-ajˆ - akˆ)
FA 4 =
( 2a )3 Two particles A and B having charges q and 2q respectively are
placed on a smooth table at separation d. A third particle C is to
® Kq 2 (- ajˆ) be clamped on the table in such a way that the particles A and B
FA5 =
a3 will be in equilibrium on the table under electrical forces only.
What should be the charge on C and where should it be clamped
® Kq 2 (-aiˆ - ajˆ) for this ?
F A6 =
( 2a ) 3
Solution
® Kq 2 (- ajˆ)
FA 7 = For the charges to be in equilibrium forces should be balanced
a3 an A on well as on B. Figure-1.25 shows the situation described
The net force experienced by A can be given as in the question.
® ® ® ® ® ® ® ® +q –Q +2q
Fnet = FA1 + FA 2 + FA3 + FA 4 + FA5 + FA 6 + FA 7
A C B
® - Kq 2 éæ 1 1 ö ù x
Þ Fnet = êç + + 1÷ (iˆ + ˆj + kˆ) ú d
a2 ëè 3 3 2 ø û
Figure 1.25

Telegram @unacademyplusdiscounts
Electrostatics 11

Forces acting on particle A are shown in figure-1.26 and given The negative value of x implies that the particle C will lie toward
as left of A at a distance ( 2 – 1)d from A (as x was measured
from A)
Kq (2 q )
FAB =
d2
For the position x = x1 = ( 2 – 1)d, Q = Q1 = – q (6 – 2)
KqQ
FAC = and for x = x2 = – d ( 2 + 1), Q = Q2 = – q (6 + 4 2 )
x2

+q Thus the two possibilities obtained above are shown in


F AB FAC figure-1.28 below
A
+q Q1 +2q
Figure 1.26

A C B
For equilibrium of A these forces are balanced so we use x1
d
FAB = FAC
or
2q Q
Þ = Q2 +q +2q
d2 x2
A A B
2qx 2 x2 d
Þ Q= … (1.4)
d2 Figure 1.28
Forces acting on particle B are shown in figure-1.27 and given
as But second possibility will not be correct for equilibrium of
both charges A and B as direction of forces are not opposite on
Kq (2 q )
FAB = A as Q2 is negative.
d2

2 Kq (Q) # Illustrative Example 1.4


FBC =
( d - x) 2
Three charges of magnitudes 5.0 × 10–7 C, – 2.5 × 10–7 C and
+2q 1.0 × 10–7 C are fixed at the three corners A, B and C of an
F BC FAB equilateral triangle of side 5.0 cm. Find the electric force on the
B
charge at vertex C due to the rest two.
Figure 1.27

Solution
For equilibrium of B these forces are balanced so we use

FAB = FBC Figure-1.29 shows the situation described in the question in


which forces acting on charge at C are also shown. These forces
2 KqQ Kq (2 q ) are given by Coulomb’s law as
Þ =
( d - x) 2 d2
Kq A qC
FAC=
Q q d2
Þ = … (1.5)
( d - x) 2 d2 9 ´109 ´ 5 ´10 -7 ´1 ´10 -7
Þ FAC =
Solving equation-(1.4) and (1.5), we get (.05) 2
Þ FAC = 0.18 N
2qx 2 q
= (d – x)2 Force on C due to B is given as
d2 d2
KqB qC
Þ 2x2 = (d – x)2 FBC =
d2
Þ 2x2 = d2 + x2 – 2xd
9 ´109 ´ -2.5 ´ 10 -7 ´ 1´ 10 -7
Þ x 2 + 2xd – d2 = 0 Þ FBC =
(.05) 2
Þ x = ( 2 – 1)d or – d (1 + 2) Þ FBC = – 0·09 N

Telegram @unacademyplusdiscounts
12 Electrostatics

FAC

q
C
–7
q3 = 1.0 × 10 C 120º

30
cm
T
30 cm Kq2
FBC F = ——2
(AB )
p q B
–––
a a = 5 cm 2 2
r mg
A q

–7
q = 5 × 10 C
(a)
a
A B
F
q1 = 5 × 10–7C q2 = – 2.5 × 10 –7C T p

q

2 +2
Figure 1.29

B
Net force on charge at C is given as p–q

® ® ®
FNet = FAC + FBC

® mg
Þ FNet = ( FAC )2 + ( FBC ) 2 + 2( FAC )( FBC ) cos(120°)
(b)
Figure 1.31
®
Þ FNet = 0.156 N
In FBD of ball B as it is in equilibrium under the influence of
three forces, we use Lami's theorem, which gives
# Illustrative Example 1.5 mg F
=
æp qö sin( p - q)
sin ç + ÷
A particle A having with a charge q = 5 × 10–7 C is clamped in a è 2 2ø
vertical wall. A small ball B of mass 100 g and having equal
charge is suspended by an insulating thread of length 30 cm mg Kq 2
Þ = q
from the wall. The point of suspension is 30 cm above the q
cos (2 ´ 0 × 30) ´ sin ´ sin q
particle A as shown in figure-1.30. Find the angle q which the 2 2
thread makes with the wall in equilibrium. Take g = 10 m/s2. mg Kq 2
Þ = q q q
q
cos (0 × 60)sin ´ 2sin cos
2 2 2 2
q q Kq 2
Þ sin2 =
2 2mg(0 × 60)
30
c

30 cm
æ 9 ´ 109 ´ ( 5 ´ 10 -7 ) 2
m

ö
æqö ç ÷
Þ sin ç ÷ = ç 2 ´ 0.1 ´ 10 ´ 0.60 ÷
B è 2ø è ø
A
Þ q = 17º

# Illustrative Example 1.6


Figure 1.30

Solution Ten charged particles are kept fied clamped on the X axis at
points x = 10 m, 20 m, 30 m, … 100 cm. The first particle has a
The forces acting on the the ball B in equilibrium are shown in charge 10–8 C, the second 8 × 10–8 C, the third 27 × 10–8 C and so
figure-1.31(a) and FBD of ball B is shown in figure-1.31(b). on. The tenth particle has a charge 100 × 10–8 C. Find the

Telegram @unacademyplusdiscounts
Electrostatics 13

magnitude of electric force acting on a 1 C charge placed at the ® ® ® ®


Kq2 q3 (r2 - r3 ) Kq1q3 ( r1 - r3 )
origin. + =0 … (1.6)
| r2 - r3 |3 | r1 - r3 |3
As the charges are collinear, we have
® ® ® ®
q1 q2 q3 q4 r2 - r3 r1 - r3
1C =–
| r2 - r3 | | r1 - r3 |
Thus from equation-(1.6), we get
q2 q1
2 =
Figure 1.32 | r2 - r3 | | r1 - r3 |2
® ® ® ®
Þ q2 ( r 1 – r 3 ) = q1 ( r 3 – r 2 )
Solution
® ®
Force on 1C charge at origin can be given by sum of forces ® q2 r1 + q1 r2
Þ r3 =
acting on it due to all the ten charges by Coulomb’s law as q1 + q2
Kq1 ´1 Kq2 ´ 1 Kq3 ´ 1 Now for equilibrium of q1, force on it due to other two charges
Fnet = -3 2
+ + +…
(10 ´ 10 ) (20 ´10-3 ) 2 (30 ´ 10-3 ) 2 must be zero so we have
® ® ® ®
K ´10 -8 é 1 + 2 + 3 +¼ 10 ö Kq1q3 ( r1 - r3 ) Kq1q2 (r2 - r1 )
3 3 3 3
Þ Fnet = ê ÷ + =0
10 -4
2
ë1 2
2
32 102 ø÷ | r1 - r3 |3 | r2 - r1 |3

Þ Fnet = 9 × 109 × 10–4 × 55 q2


Þ q3 = × |r1 – r3|2 … (1.7)
| r2 - r1 |2
Þ Fnet = 4.95 × 107 N
Substituting the value of r3 in above equation, we get
# Illustrative Example 1.7 -q1q2
q3 =
( q1 + q2 ) 2
Two positive charges q1 and q2 are located at the points with
® ®
position vectors r 1 and r 2 . Find the magnitude of a negative # Illustrative Example 1.8
®
charge q3 and the position vector r 3 of the point at which it is
to be placed for the force acting on each of the three charges to
Three small balls, each of mass 10 gm are suspended separately
be equal to zero.
from common point by silk threads, each one meter long. The
balls are identically charged and hang at the corners of an
Solution
equilateral triangle of side 0.1 metre. Find the charge on each
ball ?
Figure-1.33 shows the situation described in the question. As
already discussed for equilibrium under electrostatic forces three
Solution
charges must be collinear.
Y Figure-1.34 shows the situation described in the question. The
+q1 different forces on ball A are also shown in figure-1.34.
O

® +q3 f
r1

® l=1m
r3
+q2 C
T q
® f
r2 M
®
O X F1 A
B
q a = 0.1 m q
Figure 1.33
F ®
F2
In above situation for equilibrium of q3 force on it due to the mg
two side charges must be zero, so we have Figure 1.34

Telegram @unacademyplusdiscounts
14 Electrostatics

The force of repulsion on A due to charge q at B is given as


Kqq
F1 =
( AB ) 2
30º
Kq 2
Þ F1 = along BA … (1.8)
(0.1) 2
The force of repulsion on A due to charge q at C is given as 15º
2
Kq F
F2 = along CA … (1.9)
(0.1) 2
The force of gravity acting downward is given as
mg
W = mg
Figure 1.35
Þ W = 0.01 × 9.8 =0.098N … (1.10)
Solution
The tension T in the thread is along AO
When set up shown in figure-1.35 is in air, we have
The resultant of F1 and F2 is F which is acting along MA and it
is given as F
tan 15º = … (1.12)
® ® mg
F = F1 cos 30 + F2 cos 30 = 3 F [F = | F 1 | ¹ | F 2|]

Here M is the centroid of equilateral triangle ABC.

If we drop a perpendicular on the base of triangle ABC from O, 30º


then it passes through M. Taking moments of all the forces
about O, we have

(mg) · AM = F · OM … (1.11) F

Îr

Substituting the values, we get


æ2 ö re
mgeff = mg 1– —
(0.01 × 9.8) ç ´ 0.1´ cos 30 ÷ rs
è3 ø
Figure 1.36
1/ 2
K 3q 2 ìï 2 æ 2 ö üï
2
= × í (1) – ç ´ 0.1 ´ cos 30 ÷ ý When set up is immersed in the medium as shown in figure-1.36,
(0.1) 2 îï è3 ø þï
the electric force experienced by the ball will reduce to Finside
and we also have and the effective gravitational force will be changed to Weff given
as
OM = [(OA)2 – (AM)2]1/2
F
Finside =
Solving for q, we get q = 6.2 × 10–8 C Îr
æ r ö
Above situation can also be solved by balancing horizontal and Weff = mg ç1 - l ÷ .
and vertical forces on ball A. Students can also try with that è rs ø
method and verify that result obtained is same. Thus now after submerging in the medium, again for equilibrium
of spheres we have
# Illustrative Example 1.9 F
tan 15º =
æ r ö
Two identically charged spheres are suspended by strings of mg Îr ç 1 - l ÷
equal length. The strings make an angle of 30º with each other. è rs ø
When suspended in a liquid of density 0.8 gm/cm 3 the angle 1
Þ Îr = =2
remains same. What is the dielectric constant of liquid. Density r
1- l
of sphere = 1.6 gm/cm3. rs

Telegram @unacademyplusdiscounts
Electrostatics 15

# Illustrative Example 1.10


æ q ö
Kç ÷ Rd q q0
è 2pR ø
A ring of radius R with a uniformly distributed charge q as Þ Fe =
shown in figure-1.37. A charge q0 is now placed at the centre of R2
the ring. Find the increment in the tension in ring. Now from equation-(1.13), we can use sin (q/2) » q/2 for q/2
to be very small, we get
Kqq0 dq dq
» 2DT
2 pR 2 2
q0
Kqq0
Þ DT =
2 pR 2
q

Figure 1.37 # Illustrative Example 1.11

Solution Four small particles charged with equal positive charges Q each
are arranged at the four corners of a horizontal square of side a.
Initially when q0 was not placed there was some tension in the A unit positive charge mass m is placed at a point P, at a height
ring. This was due to the repulsion of the already present charges h above the centre of the square. What should be the magnitude
on ring. When q0 is placed the repulsion will increase. And of charge Q in order that the unit charge remain in equilibrium.
hence tension will increase. The increment in tension DT will
therefore balance the repulsion due to q0. Solution

To determine the tension increment we consider an infinitesimal The situation is shown in figure-1.39(a).
element of ring subtending angle dq at centre as shown in
figure-1.38. P
Fe F cos q
F
A B q
h
DT DT A B P F sin q
dq C
B q
dq/2 dq/2

2
DT Q Q
+ —a
DT dq
2
dq
— —
2
2 h
2
h
a

dq A
a D
Q Q A a/Ö2 O

Figure 1.38 (a) (b)

The element is now shown exaggerated. For equilibrium of this Figure 1.39
segment, we have
æ dq ö Force experienced by unit positive charge placed at P due to a
Fe = 2DT sin ç ÷ … (1.13) charge Q at vertex A is given by Coulomb’s law as
è 2 ø
Where Fe is the electric repulsion on the element due to the K (Q ´1)
charge q0 F=
æ 2 a2 ö
çç h + 2 ÷÷
Charge on element is given as è ø
q Similarly, equal forces also act on unit positive charge at P due
dq = × Rdq
2pR to charge at B, C and D. When these forces are resolved in
The electric outward force on element is given by Coulomb’s horizontal and vertical directions, the horizontal components
law as (F sin q) cancel each other due to their opposite directions and
Kq0 dq the net vertical upward force on the unit charge is added up to
Fe = 4F cos q.
R2

Telegram @unacademyplusdiscounts
16 Electrostatics

Thus net upward force on the unit charge is given as (i) Let x be the displacement of the charge +q from the mean
position. Now net force acting on the charge q toward its
4 KQ
FV = × cos q equilibrium position is
æ 2 a2 ö
çç h + ÷÷ KQq KQq
è 2 ø F= –
( a - x)2 (a + x )2
For the equilibrium of unit positive charge at P, 4 KQq ax 4 KQq ax
Þ F= » [As x << a]
F V = mg
2
(a - x ) 2 2
a4
4 KQq x
Þ
4K Q Þ F=
× cos q = mg … (1.14) a3
æ 2 a2 ö
çç h + ÷÷ After displacement restoring acceleration of particle is given as
è 2 ø

From figure-1.39(b), we have F 4KQq


a= =– x … (1.15)
m ma3
ìï h üï
In above expression we’ve included a negative sign which
cos q = í ý
ïî (h 2 + a 2 /2) ïþ shows restoring tendency of acceleration. As acceleration in
above equation-(1.15) is directly proportional to displacement
Substituting the value of cosq in equation-(1.14), we get of the particle, it verifies that particle is executing SHM. For
4 K Qh SHM of a particle, its acceleration is given as
3/2 = mg
æ 2 a2 ö a = – w2x … (1.16)
çç h + ÷÷
è 2 ø Comparing equations-(1.15) and (1.16) we get angular frequency
3/2
of SHM, given as
4 K mg æ 2 a2 ö
Þ Q= çç h + ÷÷ æ 4QqK ö
h è 2 ø w= ç ÷
è ma3 ø
The time period of SHM is given as
# Illustrative Example 1.12
2p
T1 =
Two small particles charged with equal positive charges Q each, w
are fixed apart at a distance 2a. Another small particle having a æ ma 3 ö p Î0 ma 3
charge q lies midway between the fixed charges. Show that T1 = 2p çç ÷÷ = 2p
è 4QqK ø qQ
(i) For small displacement (relative to a) along line joining the
fixed charges, the middle charge executes SHM if it is +ve and
(ii) Restoring force on – q toward its mean position is given by
(ii) For small lateral displacement, it executes SHM if it is –ve. the vertical components of the Coulombic forces on it due to
Compare the frequencies of oscillation in the two cases. the two clamped charges, given as
2 KQq
Solution F= cos q
(a 2 + x2 )
The two situations are shown in figure-1.40 in which the middle 2 KQq x
charge is displaced along the line joining or normal to it laterally. Þ F= 2 2 ·
(a + x ) (a 2 + x 2 )
+Q q x q +Q
2KQqx 2KQqx
Þ F= » [As x << a]
2
(a + x ) 2 3/ 2
a3
a (i)
After displacement restoring acceleration of particle is given as
–q
+Q x q +Q F 2KQq
a= =– x … (1.17)
m ma3
(ii)
a
In above expression we’ve included a negative sign which
shows restoring tendency of acceleration. As acceleration in
Figure 1.40 above equation-(1.17) is directly proportional to displacement

Telegram @unacademyplusdiscounts
Electrostatics 17

of the particle, it verifies that particle is executing SHM. For (iv) A charge Q is to be divided on two small objects. What
SHM of a particle, its acceleration is given as should be the value of the charges on the objects so that the
force between the objects will be maximum.
a = – w2x … (1.18)
Q Q
[ , ]
Comparing equations-(1.17) and (1.18) we get angular frequency 2 2
of SHM, given as
(v) Three charges q1, q2 and q3 are shown in figure-1.41.
æ 2QqK ö Determine the net force acting on charge q1. The charges and
w= ç ÷ separation are given as q1 = – 1.0 × 10–6 C, q2 = + 3.0 × 10–6 C,
è ma3 ø
and q3 = – 2.0 × 10–6 C, r12 = 15 cm, r13 = 10 cm and q = 30°.
Hence the time period of SHM is given as
y
2p
T2 =
w
q3
æ ma 3 ö 2p Î0 ma 3 q
T2 = 2p çç ÷÷ = 2p … (1.19) r13 r12
è 2QqK ø qQ
q1 ® x
F12 q2
Comparing the two frequencies of SHM in cases (i) and (ii) we q
have ®
F 13
n1 T1
= = 2
n2 T2
Figure 1.41
[2.64 N]

Web Reference at www.physicsgalaxy.com (vi) Three Charges of magnitude 100µC are placed at the
Age Group - Grade 11 & 12 | Age 17-19 Years corners A, B and C of an equilateral triangle of side 4m. If the
Section - ELECTROSTATICS charge at A and C are positive and the one at point B is negative,
Topic - Electrostatics what is the magnitude and direction of total force acting on
Module Number - 1 to 18 charge at C?
[5.625N]

Practice Exercise 1.1 (vii) Two negative charges of unit magnitude and a positive
charge q are placed along a straight line. At what position and
(i) Two identical small non conducting balls are charged by for what value of q will the system be in equilibrium ? Check
rubbing against each other. They are suspended from ceiling whether it is stable, unstable or neutral equilibrium.
rod through two strings of length L = 20 cm each. The separation [mid point, 0.25C]
between the suspension points being d = 5 cm. In equilibrium
the separation between the balls is r = 3 cm. Find the mass m of (viii) A charge Q is placed at each of two opposite corners of
each ball and the tension in the strings. The charge on each ball a square and a charge – q is placed at each of the remaining two
has magnitude 2 × 10–8 C. corners. If the resultant force on Q is zero, how are Q and q
[7.96g, 7.72 × 10 –2 N]
related.
[Q = –2 2 q]
(ii) Two positively charged small particles, each of mass
1.7 × 10–27 kg and carrying a charge of 1.6 × 10–19 C are placed (ix) Two balls of the same radius and weight are suspended
apart at a separation r. If each one experiences a repulsive force on threads so that their surface are in contact. A charge of
equal to its weight, find their separation. q0 = 4 × 10–7C is given to the balls which makes them repel each
[0.117 m] other and diverge to an angle of 60°. Find the mass of the balls
if the distance of balls from the point of suspension to the
(iii) Two free particles carrying charges +q and + 4q are placed centre of ball is 20cm. Find the density of the material of the
apart at a distance l. Find the magnitude, sign and location of a balls if the angle of divergence becomes 54° when the balls are
third charge which makes the system in equilibrium. immersed in kerosene of density 800kg m–3. dielectric constant
l 4q
of kerosene is er = 2
[ , ]
3 9 [1.592g, 2559kg/m 3 ]

Telegram @unacademyplusdiscounts
18 Electrostatics

(x) Two equal positive point charges are separated by a 1.3 Electric Field
distance 2a. A point test charge is located in a plane which is
normal to the line joining these charges and midway between When a charge is brought close to another fixed charge then it
them. experiences some force on it due to the presence of the fixed
(a) Calculate the radius r of the circle of symmetry in this charge. This region in surrounding of a charge where any other
plane for which the force on the test charge has a maximum charge experiences a force is called the electric field of the
value. charge. In other words we can say that electric field is the space
surrounding an electric charge q in which another charge q'
(b) What is the direction of this force, assuming a positive
experiences an electrostatic force of attraction, or repulsion.
test charge ?
[a/ 2 , radial and away from the center] Electric field is measured as a vector quantity which has a
direction and magnitude which is called strength or intensity of
(xi) Consider a fixed charge Q and another charge q is placed electric field. The direction of electric field is radially outwards
at a distance x0 from Q on a smooth plane surface. Find the in surrounding of a positive charge and is radially inwards for a
velocity of charge q as a function of x. negative charge as shown in the figure-1.43.
1
é Qq ì 1 1 ü ù 2
[ê í – ýú ]
êë 2p Î0 m î x0 x þ úû

q –q
(xii) A positive point charge 50µC is located in the plane xy at
the position vector r0 = 2iˆ + 3 ˆj , where iˆ and ĵ are the unit
vectors of the x and y axis. Find the vector of the electric field
strength E and its magnitude at the point with radius vector Electric field for a positive charge Electric field for a negative charge
r = 8iˆ – 5 ˆj . Here r0 and r are expressed in meter.. Figure 1.43
[4.5kV/m]
For electric field there are some points always to be kept in mind
(xiii) Four point charges, each of charge +q, are rigidly fixed at for understanding its applications in different situations. These
the four corners of a square planar soap film of side ‘a’. The are
surface tension of the soap film is s. If the he system of charges
and planar film are in equilibrium, then side of square is given as (1) Electric field can be defined as a space surrounding a charge
in which another ‘static’ charge experiences a force on it.
1/ N
é q2 ù
a=kê ú , find the values of k and N. (2) A moving charge can experience a force by magnetic fields
ësû also thus moving charges cannot be used to detect the presence
é 1 æ 1 öù
1/ 3 of electric field whereas magnetic field does not exert force on

p Î ç1 + ÷ú , 3] static charges.
ë 4 0 è 2 2 øû

(3) In a region of electric field if a positive charge is placed, it


(xiv) Two identical beads each having a mass m and charge q. exerts a force on the charge in the direction of electric field
When placed in a hemispherical bowl of radius R with whereas on a negative charge the direction of force is opposite
frictionless, non-conducting walls, the beads move, and at to the direction of electric field as shown in figure-1.44
equilibrium they are a distance R apart (figure-1.42). Determine
the charge on each bead.
+q1
R R
m m F
A R B
F

–q2 ®
E

Figure 1.42
1/ 2
æ 4pe 0 mgR 2 ö
[ çç ÷÷ ]
è 3 ø
Figure 1.44

Telegram @unacademyplusdiscounts

You might also like